Principle Questions - - Question 58

Ethicist: A society is just when, and only when, first, each person has an equal right to basic liberties, and second...

erojas March 24, 2018

Answer choice B

I need help understand the difference between answer choice B and D. Thanks!

Reply
Create a free account to read and take part in forum discussions.

Already have an account? log in

Mehran March 25, 2018

Hi @erojas, thanks for your post. The stimulus presents a principle, in which two separate conditions are posited as *both* sufficient and necessary ("when and only when") for a just society: (1) each person has an equal right to basic liberties, and (2) inequalities in the distribution of income and wealth are not tolerated, unless those inequalities are to everyone's advantage and are attached to jobs open to everyone.

The question stem asks you to select the answer choice that most closely conforms to this principle.

Answer choice (D) is correct. It states: Society S distributes income and wealth to everyone equally. Thus, it satisfies the second condition in the stimulus. But then we are told that Society S achieves this equal distribution of income and wealth at the expense of creating inequalities in the right to basic liberties. Thus, Society S violates the first condition in the stimulus. And so it is not a just society.

The facts presented in answer choice (B) is not necessarily inconsistent with the stimulus. It says Society S gives everyone an equal right to basic liberties, which means it conforms with the first condition in the stimulus. It then says that Society S does this at the expense of creating inequalities in the distribution of income and wealth. But the stimulus says that such inequalities may be tolerated if (a) they are to everyone's advantage and (b) are attached to jobs open to everyone. We don't know if these subconditions hold; if they do, then Society S may be just. If they do not, then Society S may not be just. Because we do not know for sure, declaring for certain that Society S "is not just" and selecting (B) is an error.

Hope this helps! Please let us know if you have any additional questions!